Blood and Heart Flashcards

1
Q

A person with an extremely high count of neutrophils is likely suffering ________.

Select one:

a. a bacterial infection
b. polycythemia
c. anemia
d. a viral infection

A

a. a bacterial infection

How well did you know this?
1
Not at all
2
3
4
5
Perfectly
2
Q

A person exhibiting suppression of immunity and clotting disorder as well as low oxygen-carrying capacity is likely suffering from which of the following?

Select one:

a. iron deficiency anemia
b. aplastic anemia
c. pernicious anemia
d. hemorrhagic anemia

A

b. aplastic anemia

How well did you know this?
1
Not at all
2
3
4
5
Perfectly
3
Q

Which body activity would be most affected if a patient lacked an adequate number of erythrocytes (anemia)?

Select one:

a. clotting
b. immune response
c. hormone transport
d. oxygen transport

A

d. oxygen transport

How well did you know this?
1
Not at all
2
3
4
5
Perfectly
4
Q

Which blood component primarily contributes to plasma osmotic pressure?

Select one:

a. electrolytes
b. albumin
c. gamma globulin
d. blood borne nutrients

A

b. albumin

How well did you know this?
1
Not at all
2
3
4
5
Perfectly
5
Q

Loss of fibrinogen within the plasma would most likely cause which of the following?

Select one:

a. fever with pain
b. pallor (pale skin)
c. edema (swelling)
d. loss of blood clotting

A

d. loss of blood clotting

How well did you know this?
1
Not at all
2
3
4
5
Perfectly
6
Q

A patient’s hematocrit shows an unusually large buffy coat. What is a likely cause for this?

Select one:

a. severe infection
b. anemia
c. lipidemia
d. thrombocytopenia

A

a. severe infection

How well did you know this?
1
Not at all
2
3
4
5
Perfectly
7
Q

With a patient that is administered an injection of erythropoietin (EPO) you would expect to see ________.

Select one:

a. decreased white blood cell count
b. increased white blood cell count
c. decreased hematocrit
d. increased hematocrit

A

d. increased hematocrit

How well did you know this?
1
Not at all
2
3
4
5
Perfectly
8
Q

With a patient who is administered an injection of colony stimulating factor (CSF), you would expect to see ________.

Select one:

a. decreased white blood cell count
b. increased white blood cell count
c. decreased red blood cell count
d. increased red blood cell count

A

b. increased white blood cell count

How well did you know this?
1
Not at all
2
3
4
5
Perfectly
9
Q

Higher viscosity of blood will increase the amount of stress placed on the heart while it is pumping. Viscosity of blood is highest when ________.

Select one:

a. hemoglobin levels are lowest
b. hematocrit is highest
c. HbA1C levels are lowest
d. plasma levels are highest

A

b. hematocrit is highest

How well did you know this?
1
Not at all
2
3
4
5
Perfectly
10
Q

Which of the following would you expect to have the least effect on hematocrit percentage?

Select one:

a. living at higher altitude
b. injection with erythropoietin (EPO)
c. prolonged or excessive fever
d. dehydration

A

c. prolonged or excessive fever

How well did you know this?
1
Not at all
2
3
4
5
Perfectly
11
Q

People that have a single allele (gene copy) for sickle cell anemia are typically not sick from the disease and are said to be carriers of sickle cell trait. These people will more often live in the malaria belt of sub-Saharan Africa. The most likely explanation for this is ________.

Select one:

a. malaria is a cause of sickle cell trait
b. the tropical climate attracts people with sickle cell trait
c. people with sickle cell trait have a better chance of surviving malaria
d. sickle cell trait is passed on to the biting mosquitoes as malaria

A

c. people with sickle cell trait have a better chance of surviving malaria

How well did you know this?
1
Not at all
2
3
4
5
Perfectly
12
Q

Lipids (either nutrients or hormones) are insoluble in water but are found traveling in the plasma of the blood. Which of the following is the most likely explanation for this?

Select one:

a. enzymes in the plasma convert lipids to soluble forms
b. lipids are carried in plasma bound to soluble plasma transport proteins
c. lipids are carried only in the lymph which is primarily composed of unsaturated fats
d. lipids are carried inside blood cells

A

b. lipids are carried in plasma bound to soluble plasma transport proteins

How well did you know this?
1
Not at all
2
3
4
5
Perfectly
13
Q

A mismatch of blood types during a transfusion is dangerous because ________.

Select one:

a. antibodies in the donor’s plasma will attack and kill the recipient’s healthy blood cells
b. clotting factors in the donor’s blood will cause unwanted clots known as thrombus
c. preformed antibodies in the recipient’s blood will bind and clump (agglutinate) the donated cells
d. white blood cells from the donor’s blood cause inflammation

A

c. preformed antibodies in the recipient’s blood will bind and clump (agglutinate) the donated cells

How well did you know this?
1
Not at all
2
3
4
5
Perfectly
14
Q

Which two factors below make rapid and substantial blood loss life threatening?

Select one:

a. loss of immunity and loss of carbon dioxide carrying capacity
b. loss of clotting ability and loss of osmotic pressure
c. loss of blood pressure and loss of oxygen carrying capacity
d. loss of immunity and loss of blood pressure

A

c. loss of blood pressure and loss of oxygen carrying capacity

How well did you know this?
1
Not at all
2
3
4
5
Perfectly
15
Q

What is the average normal pH range of blood?

Select one:

a. 8.35-8.45
b. 7.75-7.85
c. 7.35-7.45
d. 4.65-4.75

A

c. 7.35-7.45

How well did you know this?
1
Not at all
2
3
4
5
Perfectly
16
Q

Which blood type is generally called the universal donor?

Select one:

a. A positive
b. B negative
c. AB positive
d. O negative

A

d. O negative

How well did you know this?
1
Not at all
2
3
4
5
Perfectly
17
Q

Which of the following might trigger erythropoiesis?

Select one:

a. hypoxia of EPO-producing cells
b. decreased tissue demand for oxygen
c. an increased number of RBCs
d. moving to a lower altitude

A

a. hypoxia of EPO-producing cells

How well did you know this?
1
Not at all
2
3
4
5
Perfectly
18
Q

Replacing lost blood volume with an isotonic saline solution ________.
Select one:
a. restores the oxygen-carrying capacity of the circulation
b. restores the patient’s blood glucose levels
c. lowers the patient’s blood volume
d. lowers the patient’s hematocrit

A

d. lowers the patient’s hematocrit

How well did you know this?
1
Not at all
2
3
4
5
Perfectly
19
Q

Blood doping refers to the practice of ________ before an athletic event.

Select one:

a. training at high altitude to increase red blood cell production
b. injecting performance-enhancing steroids into the blood
c. removing, storing, and re-injecting a person’s red blood cells
d. injecting high amounts of oxygen into the blood stream

A

c. removing, storing, and re-injecting a person’s red blood cells

How well did you know this?
1
Not at all
2
3
4
5
Perfectly
20
Q

The cells responsible for producing platelets are called ________.

Select one:

a. megakaryocytes
b. myeloblasts
c. monoblasts
d. lymphoid stem cells

A

a. megakaryocytes

How well did you know this?
1
Not at all
2
3
4
5
Perfectly
21
Q

What organ in the body regulates erythrocyte production?

Select one:

a. kidney
b. brain
c. liver
d. pancreas

A

a. kidney

How well did you know this?
1
Not at all
2
3
4
5
Perfectly
22
Q

Normal heart sounds are caused by which of the following events?

Select one:

a. excitation of the sinoatrial (SA) node
b. closure of the heart valves
c. friction of blood against the chamber walls
d. opening of the heart valves

A

b. closure of the heart valves

How well did you know this?
1
Not at all
2
3
4
5
Perfectly
23
Q

During the isovolumetric relaxation phase of the cardiac cycle, _______.

Select one:

a. atrioventricular, aortic, and pulmonary valves are closed
b. atrioventricular valves are closed; aortic and pulmonary valves are open
c. atrioventricular valves are open; aortic and pulmonary valves are closed
d. atrioventricular, aortic, and pulmonary valves are open

A

a. atrioventricular, aortic, and pulmonary valves are closed

How well did you know this?
1
Not at all
2
3
4
5
Perfectly
24
Q

Hemorrhage with a large loss of blood causes ________.

Select one:

a. a lowering of blood pressure due to change in cardiac output
b. a rise in blood pressure due to change in cardiac output
c. no change in blood pressure but a slower heart rate
d. no change in blood pressure but a change in respiration

A

a. a lowering of blood pressure due to change in cardiac output

How well did you know this?
1
Not at all
2
3
4
5
Perfectly
25
Q

The left ventricular wall of the heart is thicker than the right wall in order to ________.

Select one:

a. accommodate a greater volume of blood
b. expand the thoracic cage during diastole
c. pump blood with greater pressure
d. pump blood through a smaller valve

A

c. pump blood with greater pressure

How well did you know this?
1
Not at all
2
3
4
5
Perfectly
26
Q

Which chambers of the heart contain oxygenated blood?

Select one:

a. right and left atria
b. right and left ventricles
c. right atrium and ventricle
d. left atrium and ventricle

A

d. left atrium and ventricle

How well did you know this?
1
Not at all
2
3
4
5
Perfectly
27
Q

If the SA node is not functioning, an ECG will show ________.

Select one:

a. higher P waves
b. more P waves than QRS waves
c. no QRS waves
d. no P waves with a HR between 40-60 bpm

A

d. no P waves with a HR between 40-60 bpm

How well did you know this?
1
Not at all
2
3
4
5
Perfectly
28
Q

The receiving chambers of the heart include the ________.

Select one:

a. right and left atria
b. right and left ventricles
c. right atrium and ventricle
d. left atrium and ventricle

A

a. right and left atria

How well did you know this?
1
Not at all
2
3
4
5
Perfectly
29
Q

The term for pain associated with deficient blood delivery to the heart that may be caused by the transient spasm of coronary arteries is ________.

Select one:

a. ischemia
b. pericarditis
c. myocardial infarct
d. angina pectoris

A

d. angina pectoris

How well did you know this?
1
Not at all
2
3
4
5
Perfectly
30
Q

To auscultate the aortic semilunar valve, you would place your stethoscope in the ________.

Select one:

a. second intercostal space to the right of the sternum
b. second intercostal space to the left of the sternum
c. fifth intercostal space inferior to the left nipple
d. fifth right intercostal space

A

a. second intercostal space to the right of the sternum

How well did you know this?
1
Not at all
2
3
4
5
Perfectly
31
Q

The source of blood carried to capillaries in the myocardium would be the ________.

Select one:

a. coronary sinus
b. fossa ovalis
c. coronary arteries
d. coronary veins

A

c. coronary arteries

How well did you know this?
1
Not at all
2
3
4
5
Perfectly
32
Q

The fact that the left ventricle of the heart is thicker than the right ventricle reveals that it ________.

Select one:

a. pumps a greater volume of blood
b. pumps blood against a greater resistance
c. expands the thoracic cage
d. sends blood through a smaller valve

A

b. pumps blood against a greater resistance

33
Q

If cardiac muscle is deprived of its normal blood supply, damage would primarily result from ________.

Select one:

a. decreased delivery of oxygen
b. a decrease in the number of available mitochondria for energy production
c. a lack of nutrients to feed into metabolic pathways
d. an inadequate supply of lactic acid

A

a. decreased delivery of oxygen

34
Q

Norepinephrine acts on the heart by ________.

Select one:

a. decreasing heart contractility
b. causing a decrease in stroke volume
c. blocking the action of calcium
d. causing threshold to be reached more quickly

A

d. causing threshold to be reached more quickly

35
Q

If the vagal nerves to the heart were cut, the result would be that ________.

Select one:

a. the heart would stop, since the vagal nerves trigger the heart to contract
b. the heart rate would increase by about 25 beats per minute
c. the atrioventricular (AV) node would become the pacemaker of the heart
d. parasympathetic stimulation would increase, causing a decrease in heart rate

A

b. the heart rate would increase by about 25 beats per minute

36
Q

The foramen ovale ________.

Select one:

a. connected the two atria in the fetal heart
b. is a condition in which the heart valves do not completely close
c. is a shallow depression in the interventricular septum
d. a connection between the pulmonary trunk and the aorta in the fetus

A

a. connected the two atria in the fetal heart

37
Q

Which vessel(s) of the heart receive(s) blood from the right ventricle?

Select one:

a. venae cavae
b. pulmonary trunk
c. aorta
d. pulmonary veins

A

b. pulmonary trunk

38
Q

Which of the following receive(s) blood during ventricular systole?

Select one:

a. aorta only
b. pulmonary arteries only
c. pulmonary veins only
d. both the aorta and pulmonary trunk

A

d. both the aorta and pulmonary trunk

39
Q

Compared to skeletal muscle, cardiac muscle ________.

Select one:

a. has gap junctions that allow it to act as a functional syncytium
b. lacks striations
c. has more nuclei per cell
d. cells are larger than skeletal muscle cells

A

a. has gap junctions that allow it to act as a functional syncytium

40
Q

During the period of ventricular filling, ________.

Select one:

a. pressure in the heart is at its peak
b. blood flows mostly passively from the atria through the atrioventricular (AV) valves into the ventricles
c. the atria remain in diastole
d. the aortic and pulmonary semilunar valves are open

A

b. blood flows mostly passively from the atria through the atrioventricular (AV) valves into the ventricles

41
Q

The second heart sound is heard during which phase of the cardiac cycle?

Select one:

a. isovolumetric relaxation
b. isovolumetric contraction
c. ventricular ejection
d. ventricular filling

A

a. isovolumetric relaxation

42
Q

Select the correct statement about cardiac output.

Select one:

a. A slow heart rate increases end diastolic volume, stroke volume, and force of contraction.
b. Decreased venous return will result in increased end diastolic volume.
c. If a semilunar valve were partially obstructed, the end systolic volume in the affected ventricle would be decreased.
d. Stroke volume increases if end diastolic volume decreases.

A

stroke volume, and force of contraction.

43
Q

Given an end diastolic volume (EDV) of 120 ml / beat and an end systolic volume (ESV) of 50 ml / beat, the stroke volume (SV) would be ________.

Select one:

a. 120 ml / beat
b. 170 ml / beat
c. 70 ml / beat
d. 50 ml / beat

A

c. 70 ml / beat

44
Q

If the mitral valve is unable to close properly, ________.

Select one:

a. blood could flow back into the left atrium
b. blood could flow back into the left ventricle
c. blood could flow back into the right atrium
d. blood could flow back into the right ventricle

A

a. blood could flow back into the left atrium

45
Q

During exercise, which of the following would occur on an electrocardiogram (ECG) compared to an individual at rest?

Select one:

a. the T wave would decrease
b. the P-R interval would decrease
c. the time from one R to the R of the next heartbeat would decrease
d. the S-T segment would decrease

A

c. the time from one R to the R of the next heartbeat would decrease

46
Q

What is the expected heart rate when a heart is removed from a living body?

Select one:

a. the heart would immediately stop beating
b. 50 beats / minute
c. 75 beats / minute
d. 100 beats / minute

A

d. 100 beats / minute

47
Q

The Frank-Starling Law states that, if other factors are constant, a ________.

Select one:

a. higher stroke volume will produce a lower end diastolic volume
b. higher end diastolic volume will produce a higher stroke volume
c. higher end diastolic volume will produce a lower stroke volume
d. higher stroke volume will produce a higher end diastolic volume

A

b. higher end diastolic volume will produce a higher stroke volume

48
Q

Reduction in the concentration of albumin in blood plasma would alter capillary exchange by ________.

Select one:

a. increasing hydrostatic pressure and edema will occur
b. decreasing colloid osmotic pressure and edema will occur
c. increasing hydrostatic pressure and blood volume, blood pressure increases
d. decreasing colloid osmotic pressure and blood volume, blood pressure increases

A

b. decreasing colloid osmotic pressure and edema will occur

49
Q

During a marathon which of the following hormones is least likely to be released by the runner?

Select one:

a. atrial natriuretic peptide (ANP)
b. epinephrine
c. angiotensin II
d. antidiuretic hormone (ADH)

A

a. atrial natriuretic peptide (ANP)

50
Q

Which structural layer of blood vessels is most responsible for maintaining blood pressure?

Select one:

a. subendothelial
b. tunica intima
c. tunica media
d. tunica externa

A

c. tunica media

51
Q

Vessels that carry blood away from the heart and are named by the organs they supply (renal) are _________.

Select one:

a. elastic arteries
b. muscular arteries
c. arterioles
d. capillaries

A

b. muscular arteries

52
Q

Which blood vessels lack elastic tissue?

Select one:

a. muscular arteries and arterioles
b. venules and veins
c. capillaries and venules
d. arterioles and capillaries

A

c. capillaries and venules

53
Q

In red bone marrow newly formed blood cells enter the circulation. You would expect to see many ________ type of capillaries in red bone marrow.

Select one:

a. sinusoid
b. continuous
c. fenestrated
d. metarterioles

A

a. sinusoid

54
Q

The aorta receives the full force of blood exiting the heart during ventricular systole. Which of the following statements best describes the adaptive anatomy of the aorta?

Select one:

a. The aorta’s tunica media is thick with dense regular connective tissue to withstand the blood’s pressure.
b. Smooth muscle is relatively thin in the aorta to increase lumen size and systemic blood flow.
c. Elastic fibers are extensive in the tunica media of the aorta and dampen the pulse pressure generated by the heart.
d. The tunica external of the aorta is nearly absent compared to other vessels.

A

c. Elastic fibers are extensive in the tunica media of the aorta and dampen the pulse pressure generated by the heart.

55
Q

Blood flow is ________ proportional to the difference in blood pressure. Blood flow is ________ proportional to the total peripheral resistance.

Select one:

a. inversely; directly
b. directly; inversely
c. inversely; inversely
d. directly; directly

A

b. directly; inversely

56
Q

Atherosclerosis causes elastic arteries to become less stretchy. How does this affect pulse pressure?

Select one:

a. Pulse pressure is chronically increased.
b. Pulse pressure is temporarily increased.
c. Pulse pressure is chronically decreased.
d. Pulse pressure is temporarily decreased.
e. Pulse pressure is unaffected by atherosclerosis.

A

a. Pulse pressure is chronically increased.

57
Q

In general, it is expected that ________.

Select one:

a. hydrostatic pressure will rise as blood moves away from the arteriole end of the capillary bed
b. hydrostatic pressure will drop as blood moves away from the arteriole end of the capillary bed
c. hydrostatic pressure will remain constant throughout the capillary bed
d. osmotic pressure will be lower in the arteriole end of the capillary bed compared to the venous end

A

b. hydrostatic pressure will drop as blood moves away from the arteriole end of the capillary bed

58
Q

Which of the following is most likely to occur in the arteriovenous shuts present in the blood vessels of the finger tips and toes?

Select one:

a. Colder temperatures will result in vasoconstriction in the arteriovenous shuts.
b. Warmer temperatures will result in vasoconstriction in the arteriovenous shuts.
c. Exercise will result in vasoconstriction in the arteriovenous shuts.
d. Colder temperatures will result in vasodilation in the arteriovenous shuts.

A

a. Colder temperatures will result in vasoconstriction in the arteriovenous shuts.

59
Q

The release of which hormone is most likely to cause a reduction in blood volume and pressure?

Select one:

a. antidiuretic hormone
b. aldosterone
c. norepinephrine
d. atrial natriuretic peptide

A

d. atrial natriuretic peptide

60
Q

Which statement best describes arteries?

Select one:

a. All carry oxygenated blood to the heart.
b. All carry blood away from the heart.
c. All contain valves to prevent the backflow of blood.
d. Only large arteries are lined with endothelium.

A

b. All carry blood away from the heart.

61
Q

Permitting the exchange of nutrients and gases between the blood and tissue cells is the primary function of ________.

Select one:

a. arterioles
b. arteries
c. veins
d. capillaries

A

d. capillaries

62
Q

Which of the following contain oxygenated blood?

Select one:

a. pulmonary trunk
b. lobar arteries
c. pulmonary veins
d. pulmonary arteries

A

c. pulmonary veins

63
Q

The pulse pressure is ________.

Select one:

a. systolic pressure plus diastolic pressure
b. systolic pressure minus diastolic pressure
c. systolic pressure divided by diastolic pressure
d. diastolic pressure plus 1/3 (systolic pressure plus diastolic pressure)

A

b. systolic pressure minus diastolic pressure

64
Q

Which of the following is likely during vigorous exercise?

Select one:

a. Blood will be diverted to the digestive organs.
b. The skin will be cold and clammy.
c. Capillaries of the active muscles will be engorged with blood.
d. Blood flow to the kidneys increases.

A

c. Capillaries of the active muscles will be engorged with blood.

65
Q

Peripheral resistance ________.

Select one:

a. decreases with increasing length of the blood vessel
b. increases as blood vessel diameter increases
c. increases as blood viscosity increases
d. is not a major factor in blood pressure in healthy individuals

A

c. increases as blood viscosity increases

66
Q

Blood flow to the skin ________.

Select one:

a. is controlled mainly by decreasing pH
b. increases when environmental temperature rises
c. increases when body temperature drops so that the skin does not freeze
d. is not an important source of nutrients and oxygen for skin cells

A

b. increases when environmental temperature rises

67
Q

Which of the following is a type of circulatory shock?

Select one:

a. hypovolemic, caused by increased blood volume
b. cardiogenic, which results from any defect in blood vessels
c. vascular, due to extreme vasodilation as a result of loss of vasomotor tone
d. circulatory, where blood volume is normal and constant

A

c. vascular, due to extreme vasodilation as a result of loss of vasomotor tone

68
Q

The form of circulatory shock known as hypovolemic shock is ________.

Select one:

a. the form of shock caused by anaphylaxis
b. any condition in which blood vessels are inadequately filled and blood cannot circulate normally
c. shock that results from large-scale loss of blood volume, or after severe vomiting or diarrhea
d. always fatal

A

c. shock that results from large-scale loss of blood volume, or after severe vomiting or diarrhea

69
Q

Which of the following is the most significant source of blood flow resistance?

Select one:

a. blood viscosity
b. total blood vessel length
c. blood vessel diameter
d. blood vessels type

A

c. blood vessel diameter

70
Q

Which of the following is least involved in pulmonary circulation?

Select one:

a. right ventricle
b. pulmonary arteries and veins
c. left atrium
d. superior vena cava

A

d. superior vena cava

71
Q

Due to the branching of arteries the type of arteries that would be most numerous would be ________.

Select one:

a. muscular arteries
b. elastic arteries
c. arterioles
d. pulmonary arteries

A

c. arterioles

72
Q

Which of the following blood pressure readings would be indicative of hypertension?

Select one:

a. 120/80
b. 140/90
c. 170/96
d. 110/60

A

c. 170/96

73
Q

A thrombus (blood clot) in the first branch of the arch of the aorta would affect the flow of blood to the ________.

Select one:

a. left side of the head and neck
b. myocardium of the heart
c. left upper arm
d. right side of the head and neck and right upper arm

A

d. right side of the head and neck and right upper arm

74
Q

Cerebral blood flow is regulated by ________.

Select one:

a. skin temperature
b. ADH
c. intrinsic autoregulatory mechanisms
d. the hypothalamic “thermostat”

A

c. intrinsic autoregulatory mechanisms

75
Q

The baroreceptors in the carotid sinus and aortic arch are sensitive to which of the following?

Select one:

a. a decrease in carbon dioxide
b. changes in arterial pressure
c. a decrease in oxygen levels
d. an increase in oxygen levels

A

b. changes in arterial pressure

76
Q

Secondary hypertension can be caused by ________.

Select one:

a. obesity
b. stress
c. kidney disease
d. smoking

A

c. kidney disease

77
Q

Where in the body would you find low oxygen levels causing vasoconstriction and high levels causing vasodilation?

Select one:

a. kidney
b. lungs
c. liver
d. heart

A

b. lungs

78
Q

If a person has lost a significant amount of blood but still maintains a normal blood pressure it does not necessarily mean that the person is maintaining adequate perfusion. Which of the following best explains why this is so?

Select one:

a. Blood vessels are constricting causing greater peripheral resistance.
b. The heart has increased its rate to increase cardiac output.
c. The heart has increased its contractile force increasing cardiac output.
d. The sympathetic nervous system has increased its nervous impulses.

A

a. Blood vessels are constricting causing greater peripheral resistance.